Are these two inverse Laplace transform solutions equivalent?

Click For Summary
SUMMARY

The discussion centers on the equivalence of two inverse Laplace transform solutions for the function Y=(8s-4)/(s²-4). One solution derived from hyperbolic functions is y=8cosh(2t)-2sinh(2t), while the instructor's solution is y=3e^(2t)+5e^(-2t). The participant confirms their solution by verifying it against the Laplace transform chart and expresses uncertainty about the equivalence of the two forms. The conclusion is that both solutions can be shown to be equivalent through expansion of hyperbolic functions into exponential terms.

PREREQUISITES
  • Understanding of inverse Laplace transforms
  • Familiarity with hyperbolic functions (sinh and cosh)
  • Knowledge of exponential functions and their properties
  • Ability to perform partial fraction decomposition
NEXT STEPS
  • Learn how to derive inverse Laplace transforms using hyperbolic functions
  • Study the relationship between hyperbolic functions and exponential functions
  • Explore partial fraction decomposition techniques in Laplace transforms
  • Practice verifying solutions through expansion of functions
USEFUL FOR

Students studying differential equations, mathematicians working with Laplace transforms, and educators teaching advanced calculus concepts.

Jrlinton
Messages
133
Reaction score
1

Homework Statement


Y=(8s-4)/(s²-4)

Homework Equations

The Attempt at a Solution


I rearranged the right side as:
8*(s/(s²-2²))-2*(2/(s²-2²))

Using the Laplace transform chart given in the class I was able to identify these as the transforms of hyperbolic sine and hyperbolic cosine making the inverse Laplace:
y=8cosh(2t)-2sinh(2t)
I checked this answer by working backwards and it all checks out but the worked out problem given by the instructor states the final answer as:
y=3e^(2t)-5e(-2t)=3[e][/2t]-5[e[/-2t]

I am wondering if these two solutions are equivalent?
 
Physics news on Phys.org
Correction: The given solution was y=3e^(2t)+5e(-2t).
I understand how he got this by using partial fractions, I would just like to know if the two functions are equivalent or if there was a mistake.
 
Jrlinton said:
Correction: The given solution was y=3e^(2t)+5e(-2t).
I understand how he got this by using partial fractions, I would just like to know if the two functions are equivalent or if there was a mistake.

You do not need to ask us; you can work it out for yourself---and you should: that is how you will learn. Just expand sinh and cosh in terms of exp and see what you get.
 
  • Like
Likes Orodruin
Question: A clock's minute hand has length 4 and its hour hand has length 3. What is the distance between the tips at the moment when it is increasing most rapidly?(Putnam Exam Question) Answer: Making assumption that both the hands moves at constant angular velocities, the answer is ## \sqrt{7} .## But don't you think this assumption is somewhat doubtful and wrong?

Similar threads

  • · Replies 1 ·
Replies
1
Views
1K
  • · Replies 10 ·
Replies
10
Views
2K
  • · Replies 1 ·
Replies
1
Views
2K
  • · Replies 8 ·
Replies
8
Views
3K
Replies
5
Views
2K
  • · Replies 7 ·
Replies
7
Views
2K
Replies
9
Views
2K
  • · Replies 9 ·
Replies
9
Views
5K
  • · Replies 2 ·
Replies
2
Views
2K
  • · Replies 1 ·
Replies
1
Views
1K